This could not be more imaginary

Algebra Level 5

Let f ( n ) = a 1 n + a 2 n + a 3 n f(n) = { a }_{ 1 }^{ n }+{ a }_{ 2 }^{ n }+{ a }_{ 3 }^{ n } .

Given that

f ( 1 ) = a 1 + a 2 + a 3 = 0 f ( 2 ) = a 1 2 + a 2 2 + a 3 2 = i f ( 3 ) = a 1 3 + a 2 3 + a 3 3 = i i { f(1)=a }_{ 1 }+{ a }_{ 2 }+{ a }_{ 3 }=0\\ f(2)={ a }_{ 1 }^{ 2 }+{ a }_{ 2 }^{ 2 }+{ a }_{ 3 }^{ 2 }=i\\ f(3)={ a }_{ 1 }^{ 3 }+{ a }_{ 2 }^{ 3 }+{ a }_{ 3 }^{ 3 }={ i }^{ i }

and that ( f ( 4 ) + f ( 7 ) ) = a + b ( e π c ) d -(f(4)+f(7))=\frac { a+b\left( { e }^{ \frac { \pi }{ c } } \right) }{ d }

Find a + b + c + d a+b+c+d given that they are integers . . . . Assumptions \textbf{Assumptions}

\bullet i = 1 i=\sqrt { -1 }

\bullet ( e ) (e) is Euler's number or ( e = lim n ( 1 + 1 n ) n ) \left(e=\lim _{ n\rightarrow \infty }{ { \left( 1+\frac { 1 }{ n } \right) }^{ n } }\right)

. . . .

Try my Other Problems


The answer is 23.

This section requires Javascript.
You are seeing this because something didn't load right. We suggest you, (a) try refreshing the page, (b) enabling javascript if it is disabled on your browser and, finally, (c) loading the non-javascript version of this page . We're sorry about the hassle.

5 solutions

Let S 1 = a 1 + a 2 + a 3 S_1=a_1+a_2+a_3 , S 2 = a 1 a 2 + a 1 a 3 + a 2 a 3 S_2=a_1a_2+a_1a_3+a_2a_3 and S 3 = a 1 a 2 a 3 S_3=a_1a_2a_3 . Using Newton's sums we obtain:

f ( 1 ) = S 1 f(1)=S_1

f ( 2 ) = S 1 2 2 S 2 f(2)=S_1^2-2S_2

f ( 3 ) = S 1 3 3 S 1 S 2 + 3 S 3 f(3)=S_1^3-3S_1S_2+3S_3

With the known values, the following system is formed;

S 1 = 0 S_1=0

S 1 2 2 S 2 = i S_1^2-2S_2=i

S 1 3 3 S 1 S 2 + 3 S 3 = i i S_1^3-3S_1S_2+3S_3=i^i

Solving that we get: S 1 = 0 S_1=0 , S 2 = i 2 S_2=-\dfrac{i}{2} and S 3 = i i 3 S_3=\dfrac{i^i}{3} .

By Newton's sums we know that f ( n ) = i 2 f ( n 2 ) + i i 3 f ( n 3 ) f(n)=\dfrac{i}{2}f(n-2)+\dfrac{i^i}{3}f(n-3) :

f ( 4 ) = i 2 f ( 2 ) + i i 3 f ( 1 ) = i 2 ( i ) + i i 3 ( 0 ) = 1 2 f(4)=\dfrac{i}{2}f(2)+\dfrac{i^i}{3}f(1)=\dfrac{i}{2}(i)+\dfrac{i^i}{3}(0)=-\dfrac{1}{2}

f ( 5 ) = i 2 f ( 3 ) + i i 3 f ( 2 ) = i 2 ( i i ) + i i 3 ( i ) = 5 i ( i i ) 6 f(5)=\dfrac{i}{2}f(3)+\dfrac{i^i}{3}f(2)=\dfrac{i}{2}(i^i)+\dfrac{i^i}{3}(i)=\dfrac{5i(i^i)}{6}

f ( 6 ) = i 2 f ( 4 ) + i i 3 f ( 3 ) = i 2 ( 1 2 ) + i i 3 ( i i ) = 4 i i + 3 i 12 f(6)=\dfrac{i}{2}f(4)+\dfrac{i^i}{3}f(3)=\dfrac{i}{2}\left(-\dfrac{1}{2}\right)+\dfrac{i^i}{3}(i^i)=-\dfrac{4i^i+3i}{12}

f ( 7 ) = i 2 f ( 5 ) + i i 3 f ( 4 ) = i 2 ( 5 i ( i i ) 6 ) + i i 3 ( 1 2 ) = 7 i i 12 f(7)=\dfrac{i}{2}f(5)+\dfrac{i^i}{3}f(4)=\dfrac{i}{2}\left(\dfrac{5i(i^i)}{6}\right)+\dfrac{i^i}{3}\left(-\dfrac{1}{2}\right)=-\dfrac{7i^i}{12}

Hence, ( f ( 4 ) + f ( 7 ) ) = 1 2 + 7 i i 12 = 6 + 7 i i 12 = 6 + 7 e π 2 12 -(f(4)+f(7))=\dfrac{1}{2}+\dfrac{7i^i}{12}=\dfrac{6+7i^i}{12}=\dfrac{6+7e^{-\frac{\pi}{2}}}{12} .

Comparing we get a = 6 a=6 , b = 7 b=7 , c = 2 c=-2 , d = 12 d=12 and a + b + c + d = 23 a+b+c+d=\boxed{23} .

Newton's sums are so tedious ;-;

Jake Lai - 6 years, 5 months ago

Log in to reply

Till now I still have no idea how this is one of my most popular problems. Personally I hated it.

Julian Poon - 5 years, 7 months ago

See my solution... although similar

Akul Agrawal - 5 years, 3 months ago

I did same.

Dev Sharma - 5 years, 5 months ago
Chew-Seong Cheong
Dec 18, 2014

Yes, I used Newton's Sums method too. It is tedious but systematic. I am attempting to make it simpler here.

Let S 1 = a 1 + a 2 + a 3 S 2 = a 1 a 2 + a 2 a 3 + a 3 a 1 S 3 = a 1 a 2 a 3 S_1 = a_1+a_2+a_3\quad S_2 = a_1a_2+a_2a_3+a_3a_1\quad S_3 = a_1a_2a_3

Then by Newton's Sums we have:

{ f ( 1 ) = S 1 = 0 f ( 2 ) = S 1 f ( 1 ) 2 S 2 = 0 2 S 2 = i S 2 = 1 2 i f ( 3 ) = S 1 f ( 2 ) S 2 f ( 1 ) + 3 S 3 = 0 0 + 3 S 3 = i i S 3 = 1 3 i i f ( 4 ) = S 1 f ( 3 ) S 2 f ( 2 ) + S 3 f ( 1 ) = 0 ( i 2 ) ( i ) + ( i i 3 ) ( 0 ) = 1 2 f ( 5 ) = i 2 f ( 3 ) + i i 3 f ( 2 ) = i 2 i i + i i 3 i = 1 2 i i + 1 + 1 3 i i + 1 = 5 6 i i + 1 f ( 7 ) = i 2 f ( 5 ) + i i 3 f ( 4 ) = i 2 ( 5 6 i i + 1 ) + i i 3 ( 1 2 ) = 1 12 ( 5 i i + 2 2 i i ) \begin {cases} f(1) = S_1 = 0 \\ f(2) = S_1f(1) - 2S_2 = 0 - 2S_2 = i \quad \quad \Rightarrow S_2 = -\frac {1}{2} i \\ f(3) = S_1f(2) - S_2f(1) + 3S_3 = 0 - 0 + 3S_3 = i^i \quad \quad \Rightarrow S_3 = \frac {1}{3} i^i \\ f(4) = S_1f(3) - S_2f(2) + S_3f(1) = 0 - (-\frac {i}{2} )(i) + (\frac {i^i}{3})(0) = -\frac {1}{2} \\ f(5) = \frac {i}{2} f(3) + \frac {i^i}{3} f(2) = \frac {i}{2} i^i + \frac {i^i}{3} i = \frac {1}{2} i^{i+1} + \frac {1}{3}i^{i+1} = \frac {5}{6}i^{i+1} \\ f(7) = \frac {i}{2} f(5) + \frac {i^i}{3} f(4) = \frac {i}{2} (\frac {5}{6}i^{i+1} ) + \frac {i^i}{3} (-\frac {1}{2}) = \frac {1}{12} (5i^{i+2} - 2i^i) \end {cases}

Therefore,

( f ( 4 ) + f ( 7 ) ) = 1 2 5 i i + 2 2 i i 12 = 6 ( 5 i i 2 i i ) 12 -(f(4)+f(7)) = \dfrac {1}{2} - \dfrac {5i^{i+2} - 2i^i} {12} = \dfrac {6 - (-5i^i-2i^i)} {12}

= 6 + 7 i i 12 = 6 + 7 ( e π 2 ) 12 \quad \quad \quad \quad \quad \quad \quad = \dfrac {6+7i^i} {12} = \dfrac {6+7 \left( e^{-\frac{\pi}{2}} \right)} {12}

a = 6 \Rightarrow a = 6 , b = 7 b=7 , c = 2 c=-2 and d = 12 a + b + c + d = 23 d = 12\quad \Rightarrow a+b+c+d = \boxed {23}

Can someone explain to me how the Newton Sums work ? I don't really seem to get it. I do get that a1 + a2 + a3 its a root, but I can't seem to make a connection. Thanks in advance

Radinoiu Damian - 6 years, 5 months ago

Log in to reply

Radinoiu,

You may want to read this: http://www.artofproblemsolving.com/Wiki/index.php/Newton%27s_Sums

Also refer to Vieta's formulas. You can also practice Vieta's formulas on Brilliant.org. It should be in Algebra section.

Chew-Seong Cheong - 6 years, 5 months ago

For simplicity lets represent a 1 = x , a 2 = y & a 3 = z a_1 = x, ~ a_2 = y ~ \& ~ a_3 = z .

Hence x + y + z = 0 x 2 + y 2 + z 2 = i x 3 + y 3 + z 3 = 3 x y z = i i = e π 2 x+y+z = 0 \\ x^2+y^2+z^2 = i \\ x^3 + y^3 +z^3 = 3xyz = i^i = e^{\frac{-\pi}{2}}

( x + y + z ) 2 = 0 x 2 + y 2 + z 2 + 2 ( x y + y z + x z ) = 0 x y + y z + x z = i 2 ( i ) (x+y+z)^2 = 0 \\ \implies x^2+y^2+z^2 + 2(xy+yz+xz) = 0 \\ \therefore xy+yz+xz = \dfrac{-i}{2} \dots \dots (i)

Now ( x y + y z + x z ) 2 = 1 4 x 2 y 2 + y 2 z 2 + z 2 x 2 + 2 x y z ( x + y + z ) = 1 4 x 2 y 2 + y 2 z 2 + z 2 x 2 = 1 4 ( i i ) (xy+yz+xz)^2 = \dfrac{-1}{4} \\ \implies x^2y^2+y^2z^2+z^2x^2 + 2xyz(x+y+z) = \dfrac{-1}{4} \\ \implies x^2y^2+y^2z^2+z^2x^2 = \dfrac{-1}{4} \dots \dots (ii)

Now ( x 2 + y 2 + z 2 ) 2 = 1 x 4 + y 4 + z 4 + 2 ( x 2 y 2 + y 2 z 2 + z 2 x 2 ) = 1 x 4 + y 4 + z 4 + 2 × ( 1 4 ) = 1 x 4 + y 4 + z 4 = f ( 4 ) = 1 2 (x^2 + y^2+z^2)^2 = -1 \\ \implies x^4 + y^4 + z^4 + 2(x^2y^2 + y^2z^2 + z^2x^2) = -1 \\ \implies x^4 + y^4 + z^4 +2\times (\dfrac{-1}{4}) = -1 \\ \therefore x^4 + y^4 + z^4 = f(4) = \dfrac{-1}{2}

Now ( x 3 + y 3 + z 3 ) ( x 4 + y 4 + z 4 ) = x 7 + y 7 + z 7 + x 3 y 3 ( x + y ) + y 3 z 3 ( y + z ) + x 3 z 3 ( x + z ) = x 7 + y 7 + z 7 ( x 3 y 3 z + y 3 z 3 x + x 3 z 3 y ) = x 7 + y 7 + z 7 x y z ( x 2 y 2 + y 2 z 2 + z 2 x 2 ) 1 2 × e π 2 = x 7 + y 7 + z 7 e π 2 × ( 1 4 ) x 7 + y 7 + z 7 = f ( 7 ) = 7 e π 2 12 (x^3+y^3+z^3)(x^4 + y^4 + z^4) = x^7 + y^7 + z^7 + x^3y^3(x+y) + y^3z^3(y+z)+x^3z^3(x+z) \\ = x^7 + y^7 + z^7 - (x^3y^3z + y^3z^3x + x^3z^3y) \\ = x^7 + y^7 + z^7 - xyz(x^2y^2+y^2z^2+z^2x^2) \\ \implies \dfrac{-1}{2} \times e^{\frac{-\pi}{2}} = x^7+y^7+z^7 - e^{\frac{-\pi}{2}} \times (\dfrac{-1}{4}) \\ \therefore x^7+y^7+z^7 = f(7) = \dfrac{-7e^{\frac{-\pi}{2}}}{12}

So ( f ( 4 ) + f ( 7 ) ) = 6 + 7 e π 2 12 a = 6 , b = 7 , c = 2 & d = 12 -(f(4)+f(7)) = \dfrac{6+7e^{\frac{-\pi}{2}}}{12} \\ \therefore ~a = 6,~b=7,~c=-2 ~ \& ~ d=12

Hence correct answer is 23 23 .

Amazing!

Julian Poon - 6 years, 2 months ago

Log in to reply

Thanks Sir.

Purushottam Abhisheikh - 6 years, 1 month ago
Akul Agrawal
Oct 31, 2015

Same thing but much less calculation than Newton's Sums

L e t z = i i ln ( z ) = i ln ( i ) ln ( z ) = i ln ( e i π 2 ) ln ( z ) = π 2 z = e π 2 \quad \quad Let\quad z\quad =\quad { i }^{ i }\\ \Rightarrow \quad \ln { (z) } =\quad i\ln { (i) } \\ \Rightarrow \quad \ln { (z) } =\quad i\ln { ({ e }^{ i\frac { \pi }{ 2 } }) } \\ \Rightarrow \quad \ln { (z) } =\quad -\frac { \pi }{ 2 } \\ \Rightarrow \quad \quad \quad z\quad =\quad { e }^{ -\frac { \pi }{ 2 } }

D G
Feb 24, 2015

There is no statement that a,b,c,d are integers, nor positive.

0 pending reports

×

Problem Loading...

Note Loading...

Set Loading...